LSAT 58 – Section 1 – Question 06

You need a full course to see this video. Enroll now and get started in less than a minute.

Target time: 1:04

This is question data from the 7Sage LSAT Scorer. You can score your LSATs, track your results, and analyze your performance with pretty charts and vital statistics - all with a Free Account ← sign up in less than 10 seconds

Question
QuickView
Type Tags Answer
Choices
Curve Question
Difficulty
Psg/Game/S
Difficulty
Explanation
PT58 S1 Q06
+LR
Method of reasoning or descriptive +Method
A
95%
164
B
1%
155
C
0%
148
D
1%
152
E
4%
159
120
129
141
+Easiest 147.03 +SubsectionMedium

The question stem reads: The argument does which of the following? This is a Method of Reasoning question.

The argument begins by stating, "When a nation is on the brink of financial collapse, its government does not violate free market principles if, in order to prevent financial collapse, it limits the extent to which foreign investors and lenders can withdraw their money." That was a mouthful, so let's break it down. We can remove the embedded clause "in order to prevent financial collapse" and add it to the end of the premise. Now we have: "The government does not violate free market principles if it limits the extent to which foreign investors and lenders can withdraw their money in order to prevent financial collapse." Ok, that makes more sense. It seems like limiting withdrawals violates the free market, so let's see what evidence they offer us. The author describes how the right to free speech does not include the right to yell fire in a crowded theater because there might be harm resulting from the "stampede" to exit the theater. The author claims that yelling fire is analogous to allowing investors to withdraw money during a financial collapse. On the author's accounts, the mad dash to withdraw money can cause just as much harm as the stampede to exit the theater. The author has made an argument by analogy. Arguably a poor analogy, but our job is not to evaluate the strength of the author's argument; it is merely to determine how the argument proceeds.

Correct Answer Choice (A) is precisely what we are looking for. When we map the stimulus to (A), we get: "tries to show that a set of principles (the free market) is limited in a specific way (limiting investors ability withdraw money during financial collapse) by using an analogy to a similar principle (free speech) that is limited in a similar way (not being allowed to yell fire in a crowded theater), precisely what we prephased.

Answer Choice (B) is incorrect. It would be difficult to map the stimulus onto this answer choice. What facts are we trying to explain? There are none, so we can ignore this answer choice.

Answer Choice (C) can be quickly crossed off because the argument contains no experimental results.

Answer Choice (D) is incorrect because the argument does not claim that a certain explanation of an observed phenomenon is wrong.

Answer Choice (E) is incorrect because there is no empirical generalization. The author makes an analogy to the limits of free speech. However, that would not be considered an empirical generalization.

Take PrepTest

Review Results

Leave a Reply